Wächst Heisenbergs Unsicherheit unter Zeitentwicklung immer?

Kürzlich gab es einige interessante Fragen zum Standard-QM und insbesondere zum Unsicherheitsprinzip, und ich habe es genossen, diese grundlegenden Konzepte zu überprüfen. Und mir wurde klar, dass ich selbst eine interessante Frage habe. Ich denke, die Antwort sollte bekannt sein, aber ich konnte das Problem nicht selbst lösen, also hoffe ich, dass es nicht ganz trivial ist.

Was wissen wir also über den Fehler der simultanen Messung unter Zeitentwicklung? Genauer gesagt ist es immer so, dass z t 0

x ( t ) 2 p ( t ) 2 x ( 0 ) 2 p ( 0 ) 2
(hier Argument ( t ) bezeichnet die Erwartung im entwickelten Zustand ψ ( t ) , oder äquivalent für Operator im Heisenberg-Bild).

Ich habe versucht, allgemeine Grenzen aus der Schrödinger-Gleichung und der Zerlegung in Energieeigenzustände usw. zu erhalten, aber ich sehe keine Möglichkeit, dies zu beweisen. Ich weiß, dass diese Aussage für ein freies Gaußsches Wellenpaket gilt. In diesem Fall erhalten wir tatsächlich Gleichheit (weil das Paket Gaußsch bleibt und weil es HUP minimiert). Ich glaube, das ist in der Tat das Beste, was wir bekommen können, und für andere Verteilungen würden wir strikte Ungleichheit erhalten.

Also, um die Fragen zusammenzufassen

  1. Stimmt die Aussage?
  2. Wenn ja, wie beweist man das? Und gibt es einen intuitiven Weg, um zu sehen, dass es wahr ist?
Warum denkst du, dass es gelten würde? Sie können auf diese Weise nicht wirklich eine Messung vornehmen (entweder Sie messen bei t = 0 oder bei t = T , aber niemals beide), also haben Sie im Grunde zwei verschiedene ψ Lösungen. Beide werden dem Prinzip unabhängig gehorchen. Verstehe ich deine Frage falsch?
Wenn Ihr Wellenpaket zunächst die Unsicherheitsgrenze sättigt (dh ein kohärenter Zustand ist), dann ist dies trivialerweise wahr - kohärente Zustände bleiben unter Zeitentwicklung kohärent. Wenn Ihr Anfangszustand kein kohärenter Zustand ist, dann ist die Evolution eindeutig stärker involviert, aber in diesem Fall könnten Sie Ihren beliebigen Anfangszustand in der kohärenten Zustandsbasis erweitern - so dass diese Ungleichung (wie sie für kohärente Zustände festgestellt wurde) immer noch verwendet werden könnte, Komponente für Komponente, um zu zeigen, dass es für den willkürlichen Zustand gilt. Oder vielleicht auch nicht. Tuck und Plug, Baby, Tuck und Plug.
@Sklivvz: Das ist kein Problem. Das Teilchen muss dennoch in jedem Moment HUP erfüllen, auch wenn Sie es nicht messen; Ich möchte diese Aussage nur quantitativ machen. Wenn es hilft, betrachten Sie dies als ein rein mathematisches Problem.
@Deepak: gute Idee. Ich weiß, dass kohärente Zustände für harmonische Oszillatoren nützlich sind, und ich nehme an, dass auch Störungen davon auftreten. Aber was ist mit einem allgemeinen System? Sind immer zusammenhängende Zustände vorhanden?
@Marek, ich verstehe, aber deine Aussage ist stärker als HUP, oder?
@Sklivvz: hm, nicht wirklich stärker. Sie ist unabhängig, gibt aber (wenn sie wahr ist) weitere Informationen über das Verhalten der Unsicherheit.
@Marek, kohärente Zustände müssen nicht "vorhanden" sein, um sie in einer basiszentrischen Berechnung zu verwenden. Denken Sie daran, dass CS eine übervollständige Basis für den Hilbert-Raum bilden.
Ich glaube nicht, dass die Aussage stimmt. Setzen Sie das Wellenpaket mit minimaler Unsicherheit auf t=0. Wie groß war die Unsicherheit vorher bei t<0? es war größer, also hat es vor t = 0 abgenommen. Allgemeiner lassen sich zeitasymmetrische Aussagen nicht aus zeitsymmetrischen Gesetzen ableiten.
@Roy: Korrigieren Sie mich, wenn ich falsch liege, aber ich bin davon ausgegangen, dass kohärente Zustände spezielle Zustände sind, die zu jeder Zeit eine bestimmte Bedingung (nämlich die Minimierung von HUP) erfüllen. Es ist offensichtlich, dass diese Bedingung vom genauen Hamilton-Operator abhängt, und es ist für mich nicht offensichtlich, dass solche Zustände immer gefunden werden können. Sprechen Sie vielleicht von CS des harmonischen Oszillators? Wenn ja, wie helfen mir diese? Sie werden sicherlich nicht durch die Evolution unter willkürlichem Hamiltonian bewahrt.
@Moshe: Es gibt Schlupflöcher in Ihrer Argumentation: Es gibt möglicherweise kein Minimum für ein bestimmtes System (nur Infimum), und wenn es ein Minimum gibt, wird es möglicherweise in der Evolution beibehalten (wie beim freien Gaußschen). Trotzdem nette Idee und ich werde versuchen, sie zu verwenden, um ein Gegenbeispiel in einem einfachen System zu finden. Was die zweite Aussage angeht: Richtig, Sie werden mir sicher sagen, dass wir auch kein zweites Gesetz erhalten können ... nur ein Scherz, ich möchte nicht in diese Diskussion einsteigen, die Boltzmann zum Selbstmord veranlasst hat :)
@Marek, in jedem Beispiel, in dem Sie die Schrödinger-Gleichung lösen können, werden Sie feststellen, dass die Menge, an der Sie interessiert sind, von t = 0 wegwächst, sowohl in die Vergangenheit als auch in die Zukunft. Dies wird durch Symmetrie garantiert. Was die allgemeine Aussage betrifft, so gilt dies auch für den zweiten Hauptsatz. Sie können ohne zusätzliche Eingaben keine zeitasymmetrischen Schlussfolgerungen aus zeitsymmetrischen Gesetzen ableiten, dies ist nur grundlegende Logik, die nichts mit Physik zu tun hat. Die ganze Diskussion dreht sich darum, was dieser zusätzliche Input ist und wo er reinkommt.
@Marek, ich habe die von Deepak vorgeschlagene Berechnung noch nicht ausprobiert (zu viele Stapel-Qs, um sie zu überprüfen), aber eine Grundlage ist genau das. Die x-Basisvektoren sind die Delta-Funktionen des Ortes, p die Delta-Basis des Impulses (mit einem gegebenen Anteil jedes Wertes für Ψ ), ebenso die CS-Basis. Das Problem ist, dass es zu vollständig ist, was zu Problemen führen kann, dh Ψ ( t ) = a 1 ( t ) C S 1 + a 2 ( t ) C S 2 + . . . .
@Moshe: gute Punkte, danke. @Roy: Ich verstehe, dass es sich um eine Reihe von Vektoren handelt, die den Hilbert-Raum überspannen, aber die Frage ist, wie sie für das allgemeine System definiert sind. Immer gleich (dh als „Eigenzustände“ von Vernichtungsoperatoren) oder abhängig im Hamiltonoperator? Ich bin ihnen außer der Standard-QM-Klasse noch nie begegnet, wenn ich über harmonische Oszillatoren spreche, daher habe ich keine Ahnung von der allgemeinen Situation.
@Marek: Es gibt viele Möglichkeiten, kohärente Zustände auf nicht harmonische Systeme zu verallgemeinern. 1) Vernichtungsoperator kohärente Zustände. 2) Kohärente Zustände dynamischer Symmetriegruppen (Nebenklasse G/H, wobei H die Stabilitätsgruppe des Bezugsvektors ist). 3) Minimale (und gleiche) Unsicherheitszustände. 4) Klauders verallgemeinerter kohärenter Zustand .... Alle Definitionen stimmen für den harmonischen Oszillator überein und können erweitert werden, um verallgemeinerte gequetschte Zustände zu erhalten. Defn 3) ist möglicherweise nicht zeitstabil, je nachdem, welche Observablen Sie minimieren. Siehe auch nLab
@Simon: sehr interessant, ich hatte keine Ahnung, dass es so viele Definitionen gibt. Können Sie auch einige nützliche Anwendungen nennen (falls Sie welche kennen)?
@Marek: Ich bin nicht wirklich jemand, der nach Anwendungen fragt ... Klauder (und andere) verwenden sie für seinen alternativen Ansatz für Pfadintegrale und Quantisierung. Das letzte Drittel von Perelemovs Buch ist physikalischen Anwendungen gewidmet. Sie können sich den Inhalt ansehen, um zu sehen, was.
@Marek: Ich habe mir dieses Zeug vor Ewigkeiten als Projekt im vierten Jahr angesehen - und die einzige Anwendung, die ich mir wirklich angesehen habe, war die Annäherung an klassische Lösungen. Wenn ich jetzt zurückblicke, verstehe ich viel mehr, und ich sehe, dass es einige Dinge gibt, die mit meiner aktuellen Arbeit zu tun haben, die ich mir wirklich genauer ansehen sollte ...
Ich persönlich denke, dass das Buch von Ali, Antoine und Gazeau eine großartige Referenz für kohärente Zustände ist. Allein die ersten paar Seiten reichen aus, um den meisten Menschen alles zu geben, was sie brauchen, um mit kohärenten Zuständen umzugehen. Als Anwendungen werden in der Kosmologie häufig gequetschte Zustände (Verallgemeinerungen kohärenter Zustände) verwendet, insbesondere um Fragen zur Entstehung der Klassik nach der Inflation zu beantworten.

Antworten (6)

Die Frage fragt nach der Zeitabhängigkeit der Funktion

f ( t ) := ψ ( t ) | ( Δ x ^ ) 2 | ψ ( t ) ψ ( t ) | ( Δ p ^ ) 2 | ψ ( t ) ,

wo

Δ x ^ := x ^ ψ ( t ) | x ^ | ψ ( t ) , Δ p ^ := p ^ ψ ( t ) | p ^ | ψ ( t ) , ψ ( t ) | ψ ( t ) = 1.

Wir werden hier das Schrödinger-Bild verwenden, in dem die Operatoren zeitlich konstant sind, während sich die Kets und BHs weiterentwickeln.

Bearbeiten : Angespornt durch Bemerkungen von Moshe R. und Ted Bunn lassen Sie uns hinzufügen, dass (unter Annahme (1) unten) die Schrödinger-Gleichung selbst unter dem Zeitumkehroperator unveränderlich ist T ^ , was ein konjugierter linearer Operator ist, so dass

T ^ t = t T ^ , T ^ x ^ = x ^ T ^ , T ^ p ^ = p ^ T ^ , T ^ 2 = 1.

Wir beschränken uns hier auf Hamiltonianer H ^ so dass

[ T ^ , H ^ ] = 0. ( 1 )

Außerdem, wenn

| ψ ( t ) = n ψ n ( t ) | n

ist eine Lösung der Schrödinger-Gleichung in einer bestimmten Basis | n , dann

T ^ | ψ ( t ) := n ψ n ( t ) | n

wird auch eine Lösung der Schrödinger-Gleichung mit einer Zeitreflexionsfunktion sein f ( t ) .

Also wenn f ( t ) zeitlich nicht konstant ist, dann können wir (möglicherweise nach einer Zeitumkehroperation) annehmen, dass es zwei Zeiten gibt t 1 < t 2 mit f ( t 1 ) > f ( t 2 ) . Dies würde der Aussage in der Ausgangsfrage widersprechen. Um das Argument zu beenden, geben wir unten ein Beispiel einer nicht konstanten Funktion f ( t ) .

Betrachten Sie einen einfachen harmonischen Oszillator Hamiltonian mit der Nullpunktsenergie 1 2 ω zur späteren Bequemlichkeit abgezogen.

H ^ := p ^ 2 2 m + 1 2 m ω 2 x ^ 2 1 2 ω = ω N ^ ,

wo N ^ := a ^ a ^ ist der Zahlenoperator.

Lassen Sie uns die Konstanten setzen m = = ω = 1 der Einfachheit halber auf eins. Dann sind die Vernichtungs- und Erzeugungsoperatoren

a ^ = 1 2 ( x ^ + ich p ^ ) , a ^ = 1 2 ( x ^ ich p ^ ) , [ a ^ , a ^ ] = 1 ,

oder umgekehrt,

x ^ = 1 2 ( a ^ + a ^ ) , p ^ = ich 2 ( a ^ a ^ ) , [ x ^ , p ^ ] = ich ,

x ^ 2 = N ^ + 1 2 ( 1 + a ^ 2 + ( a ^ ) 2 ) , p ^ 2 = N ^ + 1 2 ( 1 a ^ 2 ( a ^ ) 2 ) .

Betrachten Sie den Fock-Raum | n := 1 n ! ( a ^ ) n | 0 so dass a ^ | 0 = 0 . Betrachten Sie den Anfangszustand

| ψ ( 0 ) := 1 2 ( | 0 + | 2 ) , ψ ( 0 ) | = 1 2 ( 0 | + 2 | ) .

Dann

| ψ ( t ) = e ich H ^ t | ψ ( 0 ) = 1 2 ( | 0 + e 2 ich t | 2 ) ,

ψ ( t ) | = ψ ( 0 ) | e ich H ^ t = 1 2 ( 0 | + 2 | e 2 ich t ) ,

ψ ( t ) | x ^ | ψ ( t ) = 0 , ψ ( t ) | p ^ | ψ ( t ) = 0.

Darüber hinaus,

ψ ( t ) | x ^ 2 | ψ ( t ) = 3 2 + 1 2 cos ( 2 t ) , ψ ( t ) | p ^ 2 | ψ ( t ) = 3 2 1 2 cos ( 2 t ) ,

Weil a ^ 2 | 2 = 2 | 0 . Deswegen,

f ( t ) = 9 4 1 2 cos 2 ( 2 t ) ,

die zeitlich nicht konstant ist, und wir sind fertig. Oder alternativ können wir das Gegenbeispiel ohne die Verwendung des obigen Zeitumkehrarguments vervollständigen, indem wir einfach eine geeignete Zeitübersetzung durchführen t t t 0 .

Ich habe darüber nachgedacht, selbst ein Beispiel für einen harmonischen Oszillator auszuarbeiten (weil ich nur wenige weitere Fragen habe und es das einfachste System zu sein scheint, bei dem etwas Nichttriviales passiert), aber Sie haben mich geschlagen. Vielen Dank!
Obwohl es eine Sache gibt, die mich stört. Ich glaube, dass die Berechnung im Wesentlichen richtig ist, aber wir haben f ( 0 ) = 1 / 4 was bedeutet, dass HUP minimiert wird (es sei denn, ich missverstehe Ihre Konventionen) und daher ψ ( 0 ) müßte Gaußsch sein - ein Widerspruch zu Ihrem Anfangszustand. Ist da irgendwo ein kleiner Rechenfehler oder habe ich einen Argumentationsfehler?
Okay, ich habe es behoben (hoffe ich) :)
Lieber @Marek: Ich stimme zu, es gab Kräfte von 2 fehlt in drei Formeln.
+1 Schönes, klares, einfaches Beispiel, das den Punkt auf den Punkt bringt.
Eine Sache, die erwähnenswert ist: Sie sagen, dass die Schrödinger-Gleichung unter Zeitumkehr nicht invariant ist. Es ist wahr, dass einfach ersetzen t t ist nicht unveränderlich, sondern ändert sich gleichzeitig t t und komplexe Konjugation ψ ψ lässt die Gleichung invariant. Das heißt, für jede Lösung ψ ( t ) , gibt es eine entsprechende Lösung ψ ( t ) der zeitlich rückwärts "aussieht" (und insbesondere die gleichen Erwartungswerte für alle Operatoren hat). Das meinen die Leute, wenn sie sagen, dass die Schrödinger-Gleichung Zeitumkehrsymmetrie hat.
@TedBunn: „Sieht aus wie“ oder „ist“? Eigentlich sollte es laut Ihrer Argumentation "ist" sein. Was ist Ihre Position zu dieser Angelegenheit?

Die Schrödinger-Gleichung ist zeitsymmetrisch. Die Antwort ist daher nein.

Ich bin bei dir, aber es ist wahrscheinlich nützlich für Marek, sich selbst zu sehen, wie das an dem einfachen Beispiel funktioniert, um sich von der allgemeinen Aussage zu überzeugen.
Ja, das scheint ein gutes Argument zu sein, um die ursprüngliche Frage zu klären. Aber es wirft weitere Fragen auf :) Insbesondere Moshes Lösung (minimales Wachsen sowohl in die Zukunft als auch in die Vergangenheit) ist eine Art Sprung. Aber ich nehme an, dass die Ungleichheit auf beiden Seiten dieses Aufschwungs befriedigt wäre. Mit anderen Worten, würde die Aussage gelten, wenn wir diese einfachen Sprunglösungen und die Zeit "t = 0" zulassen würden. Oder um es klarer auszudrücken: Ich hätte eine allgemeinere Frage stellen sollen, wie die Unsicherheit als Funktion der Zeit aussieht ... Wir wissen jetzt, dass sie nicht monoton sein muss, aber vielleicht hat sie andere nette Eigenschaften.
Ich kann aus diesem Satz weder Kopf noch Zahl machen: Mit anderen Worten, würde die Aussage gelten, wenn wir diese einfachen Sprunglösungen und die Zeit "t = 0" zulassen würden. Ich weiß nicht, ob etwas Interessantes im Allgemeinen über die Zeitentwicklung von gesagt werden kann Δ x Δ p , außer natürlich, dass es unten begrenzt ist.
@Ted: ah, das war ja nicht ganz klar. Die beste Umformulierung ist wahrscheinlich diese: ob es Zeit gibt t 0 so dass die Ungleichung für alle Zeiten gilt t t 0 . Aber es ist eine andere Frage.
Ach, ich verstehe was du meinst. Das ist eine sehr plausible Vermutung. Ich wette, es stimmt für ein freies Teilchen, aber ich weiß es nicht.
Ja, ich denke, es gilt für ein freies Teilchen. Wenn Sie im Heisenberg-Bild arbeiten, können Sie das zeigen (es sei denn, ich habe die Kommutatoren vermasselt, was immer möglich ist). d ( x 2 ) / d t 2 = 2 p 2 / m 2 . Arbeiten Sie in einem Referenzrahmen, in dem p = 0 , so dass x ist konstant und kann auch gleich Null genommen werden. Dann kommt die Ungewissheit x ist nur Δ x 2 = x 2 , die eine positive zweite Ableitung hat 2 p 2 / m 2 . So beginnt es schließlich zu steigen. In dieser Situation Δ p ist konstant, also Δ x Δ p steigt schließlich an.
@Ted Bunn; Meine Intuition sagte dasselbe, aber woher weißt du das? Δ p ist konstant? Vielleicht nimmt es ab. Die Berechnung müsste ich machen. Aber es macht Sinn, dass, wenn sich die Wellenfunktion ausbreitet, sie einen kleineren durchschnittlichen Gradienten und somit einen kleineren hat Δ p .
@Ted: @Carl hat Recht. Wie ich bereits in meiner Frage geschrieben habe, minimiert das kostenlose Gaußsche Paket HUP für alle Zeiten. Aber wir wissen auch, dass die Unsicherheit in seiner Position zunimmt. Daher muss die Unsicherheit in seinem Impuls abnehmen. Ich nehme an, dies ist ein allgemeines Merkmal jeder Evolution (aber auch hier gibt es keinen Beweis, und es könnte gut sein, dass es aus trivialen Gründen fehlschlägt).
@Ted, @Carl: okay, jetzt bin ich völlig verloren. Zum einen sehe ich kein Problem mit meiner Argumentation zu HUP. Zum anderen verstehe ich Teds Kommentar über Δ p ist konstant, da der Impuls ein Bewegungsintegral für freie Teilchen ist und sich daher die Impulsverteilung des Pakets nicht mit der Zeit ändern kann. Offensichtlich muss ich etwas Triviales übersehen, aber ich kann nicht herausfinden, was es ist :(
Okay alles klar. Die weiterentwickelte Funktion ist offensichtlich nicht mehr Gaußsch. Nur seine Wahrscheinlichkeitsdichte ist, aber nicht seine Amplitude. Daher gibt es keinen Grund anzunehmen, dass es immer noch HUP minimiert.
Ich denke, dass @Marek und ich uns vollkommen einig sind. Lassen Sie mich, um explizit zu sein, die Frage von @Carl beantworten, woher wir das wissen Δ p ist konstant. Marek hat Recht: Für ein freies Teilchen gilt p n pendelt mit dem Hamiltonoperator, also alles Erwartungswerte p n sind konstant. So Δ p 2 = p 2 p 2 ist konstant. (Tatsächlich ist die gesamte Wahrscheinlichkeitsverteilung für p ist zeitlich konstant.) Infolgedessen bleibt ein Gaußsches Wellenpaket für ein freies Teilchen nicht für alle Zeiten minimale Unsicherheit. Es breitet sich im realen Raum aus, während es im Impulsraum gleich bleibt.
@Ted Bunn; Tatsächlich zeigt die Referenz, die ich gegeben habe: demonstrations.wolfram.com/EvolutionOfAGaussianWavePacket , dass der Gaussian ein Minimum hat / 2 bei t=0. Dementsprechend werde ich den Kommentar löschen, den ich gemacht habe, indem ich das Gegenteil gesagt habe.
Ich tue mich so schwer mit diesem Konzept. Ihre Aussage scheint mir falsch zu sein, und anstatt zu argumentieren, lassen Sie mich eine Gleichung aufstellen, um meine Argumentation zu untermauern. Stimmt das nicht x ( t ) 2 p ( t ) 2 x ( 0 ) 2 p ( 0 ) 2 auch? Die genaueste bekannte x und p sind bei t = 0 , also wächst mit nur dieser Information die Unsicherheit in beide Zeitrichtungen. Ja, die Gleichung ist zeitsymmetrisch, aber das stimmt damit überein, dass Ihre Antwort falsch ist.
Nun, diese Aussage kann nicht für alle Lösungen der Schrödinger-Gleichung gelten, da es nichts Besonderes gibt t = 0 in der Gleichung! Ich glaube, ich verfehle deinen Punkt.
Lassen Sie mich etwas genauer auf den Punkt meines vorherigen Kommentars eingehen. Angenommen, Sie haben eine Lösung, bei der die Unsicherheit mit zunehmender Entfernung zunimmt t = 0 in beide Richtungen. Dann durch Verschieben Ihrer (willkürlichen) Wahl von t = 0 etwas nach vorn, hättest du eine Lösung, bei der die Unsicherheit wächst t = 0 , und durch Verschieben t = 0 ein bisschen rückwärts, hätten Sie eine Lösung, bei der die Unsicherheit abfällt t = 0 .

Nein. Hier ist ein einfaches Beispiel, wo es schrumpft:

Sie haben ein Teilchen, das mit 50-prozentiger Wahrscheinlichkeit links ist und nach rechts geht, und mit 50-prozentiger Wahrscheinlichkeit, dass es rechts ist und nach links geht. Dies hat einen makroskopischen Fehler in Position und Impuls. Wenn Sie warten, bis es auf halbem Weg vorbei ist, hat es eine 100% ige Chance, in der Mitte zu sein. Dies hat einen mikroskopischen Positionsfehler. Es wird auch nur eine mikroskopische Änderung des Impulses geben. (Dabei bin ich mir nicht ganz sicher, da sich die Möglichkeiten treffen, aber wenn Sie nur vorher genau hinsehen oder sie ein wenig verfehlen lassen, funktioniert es immer noch.)

Als solches verringerte sich der Positionsfehler erheblich, aber der Impulsfehler blieb ungefähr gleich.

Mark,

Denken Sie in Begriffen von harmonischen Funktionen und ihrem Maximumprinzip (oder Mittelwertsatz).

Lassen Sie uns der Einfachheit halber (und tatsächlich ohne Verlust der Allgemeinheit) nur in Begriffen eines freien Teilchens denken, dh v ( x , j , z ) = 0 . Wenn das Potential verschwindet, ist die Schrödinger-Gleichung nichts anderes als eine Laplace-Gleichung (oder Poisson-Gleichung, wenn Sie einen Quellterm verwenden möchten). Und in diesem Fall können Sie den Mittelwertsatz (oder das Maximumprinzip) anwenden und ein Ergebnis zu Ihrer Frage erhalten: In dieser Situation sättigen Sie die Gleichheit.

Wenn Sie nun ein Potenzial haben, können Sie sich einen Laplace-Beltrami-Operator vorstellen : Alles, was Sie tun müssen, ist, das Potenzial im kinetischen Term über eine Jacobi-Metrik zu „absorbieren“ : g ~ = 2 ( E v ) g . (Beachten Sie, dass dies nur eine konforme Transformation der ursprünglichen Metrik in Ihrem Problem ist.) Und sobald dies erledigt ist, können Sie einfach die gleiche Kurbel drehen wie oben, dh wir haben das Problem auf dasselbe wie oben reduziert. ;-)

Ich hoffe, das hilft ein bisschen.

Es tut mir leid, aber ich verstehe nicht, wie dies mit Unsicherheit und Zeitentwicklung zusammenhängt. Können Sie das erklären?
@Marek: Der Punkt wurde von Qmechanic in seiner obigen Antwort explizit gemacht. Wenn Sie anwenden, was ich im Schrödinger-Bild gesagt habe, erhalten Sie sich entwickelnde Zustände, deren Größe immer an den Mittelwertsatz gebunden ist. (Wenn wir über beschränkte Operatoren sprechen, könnte dies mit ein wenig Funktionsanalyse strenger gemacht werden.)

Eine physikalische Betrachtungsweise ist, dass das Phasenraumvolumen eines Systems erhalten bleibt. Die Hamiltonsche Mechanik erhält das Volumen eines Systems auf seiner Energiefläche H = E, was in der Quantenmechanik der Schrödinger-Gleichung entspricht. Das Phasenraumvolumen auf der Energiefläche des Phasenraums setzt sich aus Volumeneinheiten zusammen 2 n für die Impuls- und Positionsvariablen plus die der Energie ich ψ / t   =   H ψ . Diese bleibt dann erhalten. Jegliches Wachsen der Ungewissheit Δ p Δ q   =   / 2 würde dann das Wachstum des Phasenraumvolumens des Systems implizieren. Dies würde dann bedeuten, dass es einen dissipativen Prozess gibt oder die Quantendynamik durch eine Hauptgleichung mit einem thermischen oder umweltbedingten Verlust in irgendeiner Form ersetzt wird. Für eine reine Einheitsentwicklung ist jedoch das Phasenraumvolumen des Systems, oder gleichbedeutend das T r ρ und T r ρ 2 sind konstant. Dies bedeutet, dass die Unsicherheitsbeziehung eine Fourier-Transformation zwischen komplementären Observablen ist, die eine Fläche erhalten   .

-1, das ist für meine Frage völlig irrelevant. Ich interessiere mich nur für reine Zustände und für diese ist das Phasenvolumen immer Null und somit trivialerweise erhalten. Dies gibt aber keine Auskunft über das Verhalten der Unsicherheit.
Das Volumen, das ein System im Phasenraum einnimmt, definiert Entropie als S   =   k   l Ö g ( Ω ) zum Ω . Die von Neumann-Entropie
S   =   k   T r   ρ l Ö g ( ρ ) .
Ein gemischter Zustand hat jedes Element von ρ   =   1 / n und die Spur ist ( 1 / n ) l Ö g ( 1 / n )   =   l Ö g ( n ) . Ein reiner Zustand nimmt dann einen Phasenraumbereich ein, der auf Einheitsvolumen normiert ist – nicht Null.

Die Heisenbergsche Unschärferelation dreht sich um den Compton-Effekt, der besagt, dass die Wellenlänge (w) umgekehrt proportional zu E, p und f ist. Wenn man jedoch mehrere quantenphysikalische Formeln sammelt, kann man diese 4 Formeln erstellen und eine Form der Geometrie verwenden, um das Unbestimmtheitsprinzip zu lösen, aber gleichzeitig den (e-) Ort und Impuls des Elektrons zu finden und gleichzeitig in der Lage zu sein, ungefähr zu bestimmen, wo es ist Präsenz war, ist und wird sein.

[Formeln stammen von mir]

  1. ((h(c/wi)-h(c/wf))/c^2)+m(e-)=m (e- Post-Photonenkollision)
  2. h(c/wi)-h(c/wf)=E(Energie, die vom Photon ausgestoßen wird, wenn es mit dem Elektron kollidiert) 3.((h(c/wi)-h(c/wf))/c)=p
  3. (h(c/wi)-h(c/wf)+m(e-)c^2=Gesamt-e-E

Unter Verwendung einer 60/30-Regel kann man den Winkel finden, in den das Elektron gestartet wird, und wo es war, bevor das Photon damit kollidierte, indem man die ursprüngliche Flugbahn und den Strom verwendet, um herauszufinden, wo es sich schneidet. Auf diese Weise können Sie seinen Standort finden.

Wenn Sie damit nicht einverstanden sind, teilen Sie dies bitte mit, damit ich es verbessern kann.

Die Impulsformel:Geben Sie hier die Bildbeschreibung ein

Die vom Photon bei Kollision ausgestoßene Energie:Geben Sie hier die Bildbeschreibung ein